Você está na página 1de 5

Lecture 1: The Real Number System

Sunil Kumar Gauttam


Department of Mathematics
LNMIIT, Jaipur
In mathematics, one begins with 1, 2, 3, these are the positive integers or the natural
numbers. We shall denote the set of positive integers by N = {1, 2, 3, }. We denote the
set of integers by Z = { , 2, 1, 0, 1, 2, }. Quotients of integers
called rational
n m are
o

numbers. We shall denote the set of all rational numbers by Q =
m, n Z, n 6= 0 .
n
Geometrically, the integers can be represented by points on a line by fixing a base point
(signifying the number 0) and a unit distance. Such a line is called the number line and it
may be drawn as in the following figure

By suitably subdividing the segment between 0 and 1, we can also represent rational numbers
1
such as , where n N, and this can, in turn, be used to represent any rational number by
n
a unique point on the number line.

You can see that the rational numbers spread themselves rather densely on this line. Now
we ask Do rationals exhaust all the number line ?. Before we start answering this let us
recall the following definition.
Definition 1.1 Given m, n Z with m 6= 0, we say that m divides n (we use notation m|n)
if n = lm for some l Z. Two integers m and n are said to be relatively prime (we use
notation (m, n) = 1) if the only integers that divide both m and n are 1 and 1.
Now consider a square whose side has unit length. Then by Pythagoras Theorem, the length
m
l of the diagonal must satisfy l2 = 2. What is l? Suppose l = , where m and n are integers.
n
1-1

1-2

Lecture 1: The Real Number System

Without loss of generality we may assume that m and n are relatively prime and n N.
Then l2 n2 = 2n2 = m2 . Hence m2 is even. Since the square of an odd integer is odd
((2k + 1)2 = 4k 2 + 4k + 1 = 2(2k 2 + 2k) + 1 for k Z), we conclude that m is even. Hence
let m = 2p for some p Z. Again 2n2 = (2p)2 = n2 = 2p2 , which shows that n is even.
This contradicts our assumption that m and n are relatively prime.

We write l = 2. Theabove discussion shows that 2 is not a rational number and we


need
numbers such as 2. Such numbers will be called irrational numbers. the number

2 can be represented by a unique point between 1 and 2 on the number line using simple
geometric constructions.

Irrational numbers are precisely the numbers needed to fill the gaps left on the number line
after marking all the rational numbers. The rational numbers and the irrational numbers
together constitute the set R, called the set of real numbers. In the light of the discussions
above we can say that every real number is represented by a unique point on the number
line. Further, every point on the number line represents one and only one real number.
Definition 1.2 Let S be a subset of R.
1. We say that S is bounded above if there exists R such that x for all x S.
Any such is called an upper bound of S. For example, S = {1, 2, 3}, {x : 0 x
3}, (, 0) are bounded above.
2. We say that S is bounded below if there exists R such that x for all x S.
Any such is called a lower bound of S.
3. The set S is said to be bounded if it is bounded aboveas well as bounded
below; other
1 1
wise, S is said to be unbounded. For example, S = 1, , , , , {x Q|x2 < 2}
2 3
are bounded.
4. An element M R is called a supremum or a least upper bound of the set S if
(a) M is an upper bound of S, that is, x M for all x S, and

Lecture 1: The Real Number System

1-3

(b) M for any upper bound of S.


It is easy to see from the definition that if S has a supremum, then it must be unique;(Let
M, M 0 R be supremum of S. First using the fact that M is an upper bound and M 0
is supremum we have M 0 M . Now using the fact that M 0 is an upper bound and M
is supremum we have M M 0 . This implies M 0 = M ) So we all the supremum of set
S (rather than a supremum of set S) we denote it by sup S.
5. An element m R is called a infimum or a greatest lower bound of the set S if
(a) m is a lower bound of S, that is, x m for all x S, and
(b) m for any lower bound of S.
For example, if S = {x R : 0 < x 1}, then inf S = 0 and sup S = 1. In this
example, inf S is not an element of S, but sup S is an element of S. So we remark
that The least upper bound or the greatest lower bound of set S may not belong to the
set S.
Exercise 1.3 Let S be a nonempty subset of R and R. If = sup S, then show that
for any  > 0, there is some x S such that  < x.
Solution: Give  > 0. Suppose there is no x S such that  < x. Then x  <
for all x S. This contradicts that is supremum.
Exercise 1.4 Let S be a nonempty subset of R and R be an upper bound. If for any
 > 0, there is some x S such that  < x then show that = sup S
Solution: Need to show that if M is any upper bound for S then M . Assume contrary,
that > M . Then for  = M > 0, there is a x S such that ( M ) < x. That is
M < x. This contradicts that M is an upper bound for S.
Exercise 1.5 Let S be a nonempty subset of R and R. If = inf S, then show that for
any  > 0, there is some x S such that +  > x.
Exercise 1.6 Let S be a nonempty subset of R and R be a lower bound. If for any
 > 0, there is some x S such that +  > x then show that = inf S
The most important, property of R that we shall assume is the following.
Completeness Property: Every nonempty subset of R that is bounded above has a supremum in R.

1-4

Lecture 1: The Real Number System

Proposition 1.7 Let S be a nonempty subset of R that is bounded below. Then S has an
infimum.
Proof: Let T = { R : a for all a S}. Since S is bounded below, T is nonempty,
and since S is nonempty, T is bounded above. Hence T has a supremum.
Claim 1.8 sup T = inf S. Let = sup T . Then x for all x S, if not then there exist
some x0 S such that x0 < . For  = x0 by Exercise 1.3, there some T such that
( x0 ) < , which means that x0 < , which is contradiction to the fact that is a
lower bound for S. Hence we proved that is a lower bound for S. Now we show that
for any lower bound of S. Which is trivial as is supremum of T , which is set of all lower
bounds of S.

Proposition 1.9 (Archimedean property) If x, y R and x > 0, then there is a positive


integer n such that nx > y.
Proof: Assume the contrary, i.e., Suppose that nx y for every positive integer n. Then
y is an upper bound of the set A = {nx : n N}. By the completeness property, let
= sup A. Note that nx for all n N implies that (n + 1)x for all n N. This
means nx x for all n N. Since x > 0, so x < . So we got an upper bound
for the set A which is strictly less than supremum. This a contradiction to the definition of
supremum.
Definition 1.10 The integer part of a real number x is denoted by [x] (greatest integer less
than or equal to x, is characterized by the following properties:
[x] Z

and

[x] x < [x] + 1.

The Archimedean property leads to the density of rationals in R and density of irrationals in R.
Proposition 1.11 Given any a, b R with a < b, there exists a rational number as well as
an irrational number between a and b.
1
1
, we can find n N such that n >
.
ba
ba
Let m = [na] + 1. Then m 1 na < m, and hence
Proof: Using Proposition 1.9 with x = 1, y =

a<

m
na + 1
1

= a + < a + (b a) = b.
n
n
n

Lecture 1: The Real Number System

1-5

m
) between a and b. Now we show that
n

given any a, b R with a < b, there exists an irrational


number
,
a
<
b
=
a+
2
<
b+

2,

b
+
2.
Then
we
claim
that
r

2
and we can find r a rational number between a + 2 and

is an irrational number between


a and b. Clearly a + 2 < r < b + 2 = a < r 2 < b.

Let us assume that r 2 is rational, i.e.


Thus we have found a rational number (namely,

m
2 = , for some m Z, n N
n
m
= r
= 2
n
r

which is contradiction as left hand side is a rational number.


Exercise 1.12 Show that Q does not have completeness property,i.e. show that there is a
non-empty subset of Q that is bounded above but does not have supremum in Q.
2
Solution:
It is clear
Let A = {r Q : r < 2}; this is a non-empty and bounded subsetof Q.
that 2 is an upper bound of set A (If b1 , b2 R with 0 b1 < b2 , then b1< b2 ). We
claim that it the supremum. Let us assume contrary, that is there exist < 2 such that
is an upper bound
for A. As is an upper bound
and 1 A so we can say that 1 .

Now 1 < 2 = r Q such that < r < 2. This implies r2 < 2 = r A.


This contradicts that is an upper bound.

Você também pode gostar